Number Theory: Structures, Examples, and Problems


II Solutions, 7. More on Divisibility



Download 1,87 Mb.
Pdf ko'rish
bet107/125
Sana08.02.2022
Hajmi1,87 Mb.
#434761
1   ...   103   104   105   106   107   108   109   110   ...   125
Bog'liq
Titu Andreescu, Dorin Andrica Number Theory Str

II Solutions, 7. More on Divisibility
Solution.
(All congruences in this problem are modulo 13.) We claim that
12
x
=
0
x
k

0 for 0

k
<
12
.
The case
k
=
0 is obvious, so suppose
k
>
0.
Note that the twelve powers 1
,
2
,
4
, . . . ,
2
11
represent all twelve nonzero resi-
dues mod 13. Thus 2
k

1
(
mod 13
)
if and only if 12
|
k
. Since the numbers
2
,
4
, . . .
, 24 are congruent (in some order) to 1
,
2
, . . . ,
12
(
mod 13
)
, we have
12
x
=
0
x
k

12
x
=
0
(
2
x
)
k
=
2
k
12
x
=
0
x
k
;
since
g
k

1, we must have
"
12
x
=
0
x
k

0. This proves our claim.
Now let
S
= {
(
x
1
, . . . ,
x
n
)
|
0

x
i

12
}
. It suffices to show that the number
of
n
-tuples
(
x
1
, . . . ,
x
n
)

S
with
f
(
x
1
, . . . ,
x
n
)

0 is divisible by 13, since
|
S
| =
13
n
is divisible by 13. Consider the sum
(
x
1
,...,
x
n
)

S
(
f
(
x
1
, . . . ,
x
n
))
12
.
This sum counts mod 13 the number of
n
-tuples
(
x
1
, . . . ,
x
n
)

S
such that
f
(
x
1
, . . . ,
x
n
)

0, since by Fermat’s little theorem,
(
f
(
x
1
, . . . ,
x
n
))
12

1
,
if
f
(
x
1
, . . . ,
x
n
)

0
,
0
,
if
f
(
x
1
, . . . ,
x
n
)

0
.
On the other hand, we can expand
(
f
(
x
1
, . . . ,
x
n
))
12
in the form
(
f
(
x
1
, . . . ,
x
n
))
12
=
N
j
=
1
c
j
n
i
=
1
x
e
j i
i
for some integers
N
,
c
j
,
e
j i
. Since
f
is a polynomial of total degree less than
n
,
we have
e
j
1
+
e
j
2
+ · · · +
e
j n
<
12
n
for every
j
, so for each
j
there exists an
i
such that
e
j i
<
12. Thus by our claim,
(
x
1
,...,
x
n
)

S
c
j
n
i
=
1
x
e
j i
i
=
c
j
n
i
=
1
12
x
=
0
x
e
j i
i

0
,
since one of the sums in the product is 0. Therefore
(
x
1
,...,
x
n
)

S
(
f
(
x
1
, . . . ,
x
n
))
12
=
(
x
1
,...,
x
n
)

S
N
j
=
1
c
j
n
i
=
1
x
e
j i
i

0
,
so the number of
(
x
1
, . . . ,
x
n
)
such that
f
(
x
1
, . . . ,
x
n
)

0
(
mod 13
)
is divisible
by 13, and we are done.


7.1. Congruences Modulo a Prime: Fermat’s Little Theorem
301
Problem 7.1.13.
Find all pairs
(
m
,
n
)
of positive integers, with m
,
n

2
, such
that a
n

1
is divisible by m for each a
∈ {
1
,
2
, . . . ,
n
}
.
(2001 Romanian International Mathematical Olympiad Team Selection Test)
Solution.
The solution is the set of all
(
p
,
p

1
)
, for odd primes
p
. The fact that
all of these pairs are indeed solutions follows immediately from Fermat’s little
theorem. Now we show that no other solutions exist.
Suppose that
(
m
,
n
)
is a solution. Let
p
be a prime dividing
m
. We first observe
that
p
>
n
. Otherwise, we could take
a
=
p
, and then
p
n

1 would not be
divisible by
p
, let alone
m
. Then because
n

2, we have
p

3, and hence
p
is
odd.
Now we prove that
p
<
n
+
2. Suppose to the contrary that
p

n
+
2. If
n
is odd, then
n
+
1 is even and less than
p
. Otherwise, if
n
is even, then
n
+
2 is
even and hence less than
p
as well, because
p
is odd. In either case, there exists
an even
d
such that
n
<
d
<
p
with
d
2

n
. Setting
a
=
2 and
a
=
d
2
in the given
condition, we find that
d
n

2
n
d
2
n

1
·
1

1
(
mod
m
),
so that
d
n

1

0
(
mod
m
)
as well. Because
n
<
d
<
p
<
m
, we see that
1
,
2
, . . . ,
n
,
d
are
n
+
1 distinct roots of the polynomial congruence
x
n

1

0
(
mod
p
)
. By Lagrange’s theorem, however, this congruence can have at most
n
roots, a contradiction.
Thus, we have sandwiched
p
between
n
and
n
+
2, and the only possibility is
that
p
=
n
+
1. Therefore, all solutions are of the form
(
p
k
,
p

1
)
with
p
an odd
prime. It remains to prove that
k
=
1. Using
a
=
n
=
p

1, it suffices to prove
that
p
k
((
p

1
)
p

1

1
).
Expanding the term
(
p

1
)
p

1
modulo
p
2
, and recalling that
p
is odd, we
have
(
p

1
)
p

1
=
p

1
i
=
0
p

1
i
(

1
)
p

1

i
p
i

p

1
0
(

1
)
p

1
+
p

1
1
(

1
)
p

2
p

1

p
(
p

1
)

p
+
1

1
(
mod
p
2
).
It follows immediately that
k
cannot be greater than 1, completing the proof.


302
II Solutions, 7. More on Divisibility
Problem 7.1.14.
Let p be a prime and b
0
an integer,
0
<
b
0
<
p. Prove that
there exists a unique sequence of base- p digits b
0
,
b
1
,
b
2
, . . . ,
b
n
, . . .
with the
following property: If the base- p representation of a number x ends in the group
of digits b
n
b
n

1
. . .
b
1
b
0
, then so does the representation of x
p
.
Solution.
We are looking for a sequence
b
0
,
b
1
,
b
2
, . . . ,
b
n
, . . .
of base
p
digits
such that the numbers
x
n
=
b
0
+
b
1
p
+ · · · +
b
n
p
n
and
x
p
n
are congruent mod-
ulo
p
n
+
1
for each
n
=
0
,
1
,
2
, . . .
Of course, the choice of the first term
b
0
is
predetermined, and given in the problem statement; let us note that the numbers
x
0
=
b
0
and
x
p
0
are congruent modulo
p
by Fermat’s little theorem. Suppose that
the base
p
digits
b
1
,
b
2
, . . . ,
b
n
are already chosen in such a way that
x
p
n

x
n
(
mod
p
n
+
1
)
. We shall prove that there is a unique digit
b
n
+
1
such that
(
x
n
+
b
n
+
1
p
n
+
1
)
p

x
n
+
b
n
+
1
p
n
+
1
(
mod
p
n
+
2
)
;
this proves the existence and the uniqueness at the same time. Since
(
x
n
+
b
n
+
1
p
n
+
1
)
p
=
x
p
n
+
p
1
x
p

1
n
b
n
+
1
p
n
+
1
+
C p
n
+
2
for some integer constant
C
, and since
p
1
is divisible by
p
, we get
(
x
n
+
b
n
+
1
p
n
+
1
)
p

x
p
n
(
mod
p
n
+
2
).
Hence
b
n
+
1
should satisfy the congruence
x
p
n

x
n

b
n
+
1
p
n
+
1

0
(
mod
p
n
+
2
).
(
1
)
By the induction hypothesis, the number
x
p
n

x
n
is divisible by
p
n
+
1
. This
implies that its
(
n
+
2
)
nd base
p
digit (from right to left) is indeed the only choice
for
b
n
+
1
such that (1) holds. The inductive proof is complete.
Problem 7.1.15.
Determine all integers n
>
1
such that
2
n
+
1
n
2
is an integer.
(31st International Mathematical Olympiad)
Solution.
We will prove that the problem has only the solution
n
=
3. First,
observe that
n
is an odd number. Then, we prove that 3
|
n
.
Let
p
be the least prime divisor of
n
. Since
n
2
|
2
n
+
1
,
2
n
+
1

0
(
mod
p
)
and 2
2
n

1
(
mod
p
)
. By Fermat’s little theorem, 2
p

1

1
(
mod
p
)
. Then
2
d

1
(
mod
p
)
, where
d
=
gcd
(
p

1
,
2
n
)
. By the definition of
p
,
d
has no
prime divisor greater than 2, which shows that
d
=
2. It follows that
p
=
3.
Let
n
=
3
k
m
, where
k

1 and
(
3
,
m
)
=
1. Using the identity
x
3
k
+
1
=
(
x
+
1
)(
x
2

x
+
1
)(
x
2
·
3

x
3
+
1
)
· · ·
(
x
2
·
3
k

1

x
3
k

1
+
1
)



Download 1,87 Mb.

Do'stlaringiz bilan baham:
1   ...   103   104   105   106   107   108   109   110   ...   125




Ma'lumotlar bazasi mualliflik huquqi bilan himoyalangan ©hozir.org 2024
ma'muriyatiga murojaat qiling

kiriting | ro'yxatdan o'tish
    Bosh sahifa
юртда тантана
Боғда битган
Бугун юртда
Эшитганлар жилманглар
Эшитмадим деманглар
битган бодомлар
Yangiariq tumani
qitish marakazi
Raqamli texnologiyalar
ilishida muhokamadan
tasdiqqa tavsiya
tavsiya etilgan
iqtisodiyot kafedrasi
steiermarkischen landesregierung
asarlaringizni yuboring
o'zingizning asarlaringizni
Iltimos faqat
faqat o'zingizning
steierm rkischen
landesregierung fachabteilung
rkischen landesregierung
hamshira loyihasi
loyihasi mavsum
faolyatining oqibatlari
asosiy adabiyotlar
fakulteti ahborot
ahborot havfsizligi
havfsizligi kafedrasi
fanidan bo’yicha
fakulteti iqtisodiyot
boshqaruv fakulteti
chiqarishda boshqaruv
ishlab chiqarishda
iqtisodiyot fakultet
multiservis tarmoqlari
fanidan asosiy
Uzbek fanidan
mavzulari potok
asosidagi multiservis
'aliyyil a'ziym
billahil 'aliyyil
illaa billahil
quvvata illaa
falah' deganida
Kompyuter savodxonligi
bo’yicha mustaqil
'alal falah'
Hayya 'alal
'alas soloh
Hayya 'alas
mavsum boyicha


yuklab olish